LSAT and Law School Admissions Forum

Get expert LSAT preparation and law school admissions advice from PowerScore Test Preparation.

 Administrator
PowerScore Staff
  • PowerScore Staff
  • Posts: 8927
  • Joined: Feb 02, 2011
|
#23111
Complete Question Explanation

Method of Reasoning. The correct answer choice is (D)

The argument introduces the principle that people should be rewarded according to effort, and then concludes that the principle is a very bad idea. The reason the principle is called a bad idea is that under such a principle the people with the least ability would be given the most incentive to do a particular thing.

You are asked to identify the argumentative strategy, so you should focus on the fact that the argument uses the possible outcome of using the principle to attack the validity of the principle.

Answer choice (A) The argument attacks the principle, and does not promote it.

Answer choice (B) This choice might seem very attractive, because it discusses undesirable outcomes. However, this choice is wrong, because it incorrectly suggests that the argument includes actual evidence, and the stimulus is decidedly lacking in anything even resembling evidence. Furthermore, the wording of the choice suggests that the argument discusses a specific case, which it does not.

Answer choice (C) The argument demonstrates nothing, as it is merely a battle of unconfirmed possibilities. Furthermore, the argument proposes a negative consequence of the principle, but does not specifically address any assumed positive consequence. Even though you might assume, for instance, that the principle's intended benefit was the implied specific kind of fairness or task-allocation, and that the argument attacks the possibility of that outcome, you should not select a choice on that basis. The principle's intent could very well be different from the intent the argument implies.

Answer choice (D): This is the correct answer choice. The argument attempts to reject the principle on the grounds that the principle has a possibly undesirable outcome. You should notice that the word "attempting" makes this choice better, because in fact the argument does not successfully defeat the principle. There is no reason to believe that the intended outcome of the principle was the implied specific version of fairness or task-allocation, and there is no reason to believe that since the unskilled had the most incentive, the incentive for other, skilled persons would be insufficient to fill jobs without stooping to hire the incompetent.

Answer choice (E) The argument does not show that it would be impossible to apply the principle, only that some people might find it undesirable to apply the principle.
 lizzieb
  • Posts: 4
  • Joined: Apr 13, 2015
|
#18527
For this question I chose answer E and in reviewing the correct answer choice, D, I still can't really understand why D is a better choice than E. I can see that it is correct but I don't understand why D is not also correct.
Thanks!!
 Andrew Ash
PowerScore Staff
  • PowerScore Staff
  • Posts: 32
  • Joined: Sep 15, 2014
|
#18533
Hi Lizzie,

This is a Method of Reasoning question, and like all questions in the First Family, you can always check your answer choice by using the Fact Test: every single element that's in your answer choice must also be present in the stimulus.

Answer choices (D) and (E) both contain the concept of a theoretical principle that doesn't work in practice, which is what we're looking for. Answer choice (E) runs off the rails, however, when it says that the principle could not be "uniformly applied" in practice. That implies that the principle would work sometimes and not others, but actually, the stimulus claims that the principle would always work badly. So, this idea of a principle that cannot be "uniformly applied" is a new element that wasn't present in the stimulus, which allows us to eliminate answer choice (E).

I hope this helps. I'm going to address your other questions in a bit!

Thanks,
Andrew
 Blueballoon5%
  • Posts: 156
  • Joined: Jul 13, 2015
|
#45379
Hi! I got the right answer, but I hope someone can help me understand the meaning of answer choice C. The answer choice C reads: "demonstrating that a consequence that had been assumed to follow from the principle under consideration need not follow from it." I don't understand what that sentence is saying. Is is saying that the supposed consequence doesn't actually have to happen?
 Adam Tyson
PowerScore Staff
  • PowerScore Staff
  • Posts: 5153
  • Joined: Apr 14, 2011
|
#49566
You got it, blueballon! That is exactly what that means!

Get the most out of your LSAT Prep Plus subscription.

Analyze and track your performance with our Testing and Analytics Package.